GMAT Data Sufficiency Discussions

If X and Y are positive integers, is X>Y?

(1) X^2Suff
Option 1) sqrX
InSuff
ANS : A
Is xy
1). (X^3 * Y^5)/ (X*Y^2) 2). X - |y|
Option 1) x^2 * y^3 inSuff

Option 2) x|-|y| Suff

ANS : B

If X and Y are positive integers, is X>Y?

(1) X^2Suff
Option 1) sqrX
InSuff
ANS : A
Is xy
1). (X^3 * Y^5)/ (X*Y^2) 2). X - |y|
Option 1) x^2 * y^3 inSuff

Option 2) x|-|y| Suff
ANS : B

The answer is not B.


Please check your calc again??

In the xy plane at what points does the graph y= (x+a)(X+b) intersect teh x- axis?

a. a+b = -1
b. the graph intersect the y-axis at (0, -6)

1)If k is a positive integer, then 20k is divisible by how many different positive integers?
a.k is prime
b.k is 7

1)If k is a positive integer, then 20k is divisible by how many different positive integers?
a.k is prime
b.k is 7


20k = 2^2*5*k

If N = a^m*b^n*c^p *...

No of factors = (m+1)(n+1)(p+1).....

St 1 : Not suff ...depends whether k is 2, 5 or some other prime no ...

St 2 : One def value of k . Hence one def no, hence def no of factors .. Suff

Ans B
In the xy plane at what points does the graph y= (x+a)(X+b) intersect teh x- axis?

a. a+b = -1
b. the graph intersect the y-axis at (0, -6)


Ans C ...

Interesting, u only had posted the explanation for this one few days back ...so i'll save some time by not posting the explanation again

Here is the explanation : http://www.pagalguy.com/discussions/gmat-data-sufficiency-discussions-25020702
In the xy plane at what points does the graph y= (x+a)(X+b) intersect teh x- axis?

a. a+b = -1
b. the graph intersect the y-axis at (0, -6)


My take C.

1.a+b =-1
-does not signify anything.

2. intercept at (0,-6).
-this gives ab=-6.

two options : (-6,1 & -3 2)

hence no particular solution.

Combining both the conditions, we can confirm that the point is (-3,2 ).

PG Rocks !
Guys,

To round off numbers in GMAT, which rule is to be followed from the following:
PS: Just mentioning the one i'm doubtful about.

1.9365 rounded off to 100th digit becomes 1.936

or

1.9335 rounded off to 100th digit becomes 1.934

Hope you understand what am i trying to clear here!
In case you don't, here it is:

When digit to be rounded off is 5, look for the digit to the left of it. If thats even, drop 5 and leave the leave the digit to the left unchanged.
(for 1st eg)

If thats odd, drop 5 and increment the previous digit by 1.
(for 2nd eg)

Please help!


I believe the second approach is good ( though i have not worked much on the gmat maths). This is what we were taught in the Kendriya Vidyalayas .
IMO A

take1) W/M+W = W/2.05W so 1) Alone is suff.

take 2) onlly percentages present no information on the actual demographics.

So A is suff.


Most of the people are goinbg with answer A.
But since anything isn't explicitly mentioned, can we miss the possibility of eunuchs? And hence shouldn't the answer be E?? :drinking:

E for the second question.

1. (X^3 * Y^5)/ (X*Y^2) 2. X - |y| if x=1 and y=4,
-3
if x= -1 and y =4
-3
both "same sign" and "diff sign" satisfied the inequality hence insuff.

If X and Y are positive integers, is X>Y?
(1) X^2
1). (X^3 * Y^5)/ (X*Y^2) 2). X - |y|

C
2 equations and 2 unknowns

In the xy plane at what points does the graph y= (x+a)(X+b) intersect teh x- axis?

a. a+b = -1
b. the graph intersect the y-axis at (0, -6)
Ans C ...

Interesting, u only had posted the explanation for this one few days back ...so i'll save some time by not posting the explanation again

Here is the explanation : http://www.pagalguy.com/discussions/gmat-data-sufficiency-discussions-25020702



This thought had occurred to me, but posted the ques anyway.

Is ab positive?
(1) (a+b)2 (2) a = b

D

Take 2) here there is no indication of the sign; But since both are +ve or both are -ve the product is always +ve. So suff.

Take 1) let a= -1 and b=2
so we have 1
now take a=2 and b =-1 again the same result and product -ve.

now take a=1 and b=2 here the inequlaity is not satisfied so the values cannot be +ve

now take a=-1 and b=-2 the inequality is not satisfies so the values cannot be both -ve.

So the to satify the ineq the value of ab must always be -ve.
The ans is D


siddharthaduggirala Says
See my explanation in red.


Hey Siddharth .

Agree with u on St 1

St 2 : what if a=b=0 then ab is not positive , for any other equal values of a and b, ab is positive ..NS

IMO ..Ans A

Correct. A is the OA.

Hey Siddharth .

Agree with u on St 1

St 2 : what if a=b=0 then ab is not positive , for any other equal values of a and b, ab is positive ..NS

IMO ..Ans A
Hey Siddharth .

Agree with u on St 1

St 2 : what if a=b=0 then ab is not positive , for any other equal values of a and b, ab is positive ..NS

IMO ..Ans A



Thanks for pointing the mistake. i completely missed out on this point.

Car X leaves town A at 2.00pm and drives toward town B at a constant rate of m miles/hr. 15 mins later, car Y begins driving from town B to town A at a constant rate of n miles/hr. If both X and Y drive along the same route, will car X be closer to town A or town B when it passes the car Y?

1. Car X arrives in town B 90 mins after leaving city A
2. Car Y arrived in town A at the same time car X arrived in town B.

In the xy plane at what points does the graph y= (x+a)(X+b) intersect teh x- axis?

a. a+b = -1-- insuff
b. the graph intersect the y-axis at (0, -6)--insuff


usin both the options we can find the values
hence
Car X leaves town A at 2.00pm and drives toward town B at a constant rate of m miles/hr. 15 mins later, car Y begins driving from town B to town A at a constant rate of n miles/hr. If both X and Y drive along the same route, will car X be closer to town A or town B when it passes the car Y?

1. Car X arrives in town B 90 mins after leaving city A
2. Car Y arrived in town A at the same time car X arrived in town B.


My take C.It would be closer to B.

let the distance travelled by X in the initial 15 mins be l.
total distance between A & B =6l

=> since both the cars meet reach the respective destinations at 3:30
we can find the ratio between m & n
5n=6m

from this we can find out the meeting point in terms of l...which comes out to be 36l/11...which is greater than 3l ...hence it is nearer/closer to city B.

( hopefully,i have not made yet another silly mistake )

PG Rocks
i concur !!
whats the rite answer
but i ve read in some books tht although its not necessary for the two answer to be the same(calculated frm the two separate options) but a question has never been cited to give two different answers.
i also faced a couple of similar questions whr i marked D as the answer but at the end found myself to be wrong...ans was either A or B
but thrs no flaw in the reasoning given above..
plz explain......

The flaw in reasoning is that we have not considered 6 and 10 for the second option. The LCM of 6 & 10 is 30 and when 6 divides 10, it leaves out 4 which is greater than 1. Hence, the second option is insufficient. According to me, the answer should be A. Is it?
Car X leaves town A at 2.00pm and drives toward town B at a constant rate of m miles/hr. 15 mins later, car Y begins driving from town B to town A at a constant rate of n miles/hr. If both X and Y drive along the same route, will car X be closer to town A or town B when it passes the car Y?

1. Car X arrives in town B 90 mins after leaving city A
2. Car Y arrived in town A at the same time car X arrived in town B.


From 1) in 90 mins car travels: M + M/2 miles
From 2) in 75 mins car B travels: N + N/4 miles

As the distance is same:

3M/ 2 = 5N / 4

6M = 5N ==> M